Đến nội dung

xuanhoan23112002 nội dung

Có 95 mục bởi xuanhoan23112002 (Tìm giới hạn từ 29-04-2020)



Sắp theo                Sắp xếp  

#709395 Đề thi vào trường chuyên Thái Bình năm 2019 (vòng 2)

Đã gửi bởi xuanhoan23112002 on 27-05-2018 - 20:34 trong Tài liệu - Đề thi

Câu 6:

Ta có:$\frac{1}{\sqrt{5a^2+2ab+2b^2}}=\frac{1}{\sqrt{(2a+b)^2+(a-b)^2}}\leq \frac{1}{2a+b}\leq \frac{1}{9}(\frac{2}{a}+\frac{1}{b})$ (bất đẳng thức Schwarz)

Chứng minh tương tự như trên ta có:

$P\leq \frac{1}{3a}+\frac{1}{3b}+\frac{1}{3c}$

Ta cũng có:$\frac{1}{a}+\frac{1}{b}+\frac{1}{c}\leq \sqrt{3(\frac{1}{a^2}+\frac{1}{b^2}+\frac{1}{c^2})}=\sqrt{3}$ (bất đẳng thức AM-GM)

Từ đó ta có: $P\leq \frac{\sqrt{3}}{3}$

Đẳng thức xảy ra $\Leftrightarrow a=b=c=\sqrt{3}$

Vậy MaxP = $\frac{\sqrt{3}}{3}\Leftrightarrow a=b=c=\sqrt{3}$.




#709350 Đề thi vào 10 chuyên Lê Hồng Phong - Nam Định 2018 - 2019 - Toán Chuyên

Đã gửi bởi xuanhoan23112002 on 27-05-2018 - 10:49 trong Tài liệu - Đề thi

Câu 5:

a. Có $\frac{1}{2}\sqrt{(a+3b)(b+3a)}\leq \frac{a+3b+b+3a}{4}=a+b$ (bất đẳng thức AM-GM)

Từ giả thiết: $\sqrt{a}+\sqrt{b}=1$ 

Bình phương 2 vế ta có: $2\sqrt{ab}=1-a-b$ 

Hay $4ab=(1-a-b)^2$

Nên bất đẳng thức cần chứng minh tương đương với:

$3(a+b)^2+(1-a-b)^2\geq 2(a+b)$

$\Leftrightarrow (2a+2b-1)^2\geq 0$ (luôn đúng)

Vậy bất đẳng thức đã cho được chứng minh.




#710109 Đề thi tuyển sinh vào lớp 10 THPT tỉnh Bắc Giang năm học 2018-2019

Đã gửi bởi xuanhoan23112002 on 06-06-2018 - 15:07 trong Tài liệu - Đề thi

Câu 5: Ta có:

$P=\frac{81x^2+18225x+1}{9x}-\frac{6\sqrt{x}+8}{x+1}\geq \frac{18x}{9x}-\frac{9x+9}{x+1}+2025= 2018$ (bất đẳng thức Cauchy)

Đẳng thức xảy ra $\Leftrightarrow x=\frac{1}{9}> 0$

Vậy $MinP=2018\Leftrightarrow x=\frac{1}{9}$




#710186 Đề thi tuyển sinh vào lớp 10 THPT chuyên tỉnh Vĩnh Phúc năm học 2018-2019

Đã gửi bởi xuanhoan23112002 on 07-06-2018 - 11:41 trong Tài liệu - Đề thi

Câu 3 (phỏng theo lời giải của thầy Võ Quốc Bá Cẩn) 

Ta có: $\frac{a^2}{a^2+ab+b^2}+\frac{c^2}{c(a+b+c)}\geq \frac{(a+c)^2}{a^2+b^2+c^2+ab+bc+ca}$ (bất đẳng thức Schwarz)

Làm tương tự với 3 phân thức còn lại ta có:

$\frac{a^2}{a^2+ab+b^2}+\frac{b^2}{b^2+bc+c^2}+\frac{c^2}{c^2+ca+a^2}\geq 1$

Đẳng thức xảy ra $\Leftrightarrow a=b=c> 0$

Vậy ta có điều phải chứng minh.




#710048 Đề thi tuyển sinh vào lớp 10 THPT chuyên tỉnh Vĩnh Phúc năm học 2018-2019

Đã gửi bởi xuanhoan23112002 on 05-06-2018 - 19:45 trong Tài liệu - Đề thi

Câu 5:

Từ giả thiết ta có: $c=a+b-\sqrt{ab}$

$P=\frac{c^2}{ab}+\frac{c^2}{a^2+b^2}+\frac{\sqrt{ab}}{a+b}$

$P\geq c^2(\frac{1}{2ab}+\frac{1}{2ab}+\frac{1}{a^2+b^2})+\frac{\sqrt{ab}}{a+b}\geq \frac{9c^2}{(a+b)^2+2ab}+\frac{\sqrt{ab}}{a+b} \geq \frac{6(a+b-\sqrt{ab})^2}{(a+b)^2}+\frac{\sqrt{ab}}{a+b}=6-\frac{11\sqrt{ab}}{a+b}+\frac{6ab}{(a+b)^2}=6(\frac{\sqrt{ab}}{a+b}-\frac{1}{2})^2-\frac{5\sqrt{ab}}{a+b}+\frac{9}{2}$ $\geq -\frac{5}{2}+\frac{9}{2}=2$ ( theo các BĐT AM-GM và Schwarz)

 Đẳng thức xảy ra $\Leftrightarrow a=b=c>0$

Vậy $MinP=2\Leftrightarrow a=b=c>0$




#710107 Đề thi tuyển sinh vào lớp 10 chuyên toán tỉnh Kiên Giang

Đã gửi bởi xuanhoan23112002 on 06-06-2018 - 14:53 trong Tài liệu - Đề thi

Bài 7: Bài bất đẳng thức có vẻ dễ nhỉ

Ta có:

$\frac{x^2}{y+z}+\frac{y^2}{z+x}+\frac{z^2}{x+y}\geq \frac{(x+y+z)^2}{2(x+y+z)}= \frac{x+y+z}{2}=1$ ( bất đẳng thức Schwarz)

Đẳng thức xảy ra $\Leftrightarrow x=y=z=\frac{2}{3}$

Vậy ta có điều phải chứng minh.




#710076 Đề thi tuyển sinh vào chuyên Lê Quý Đôn tỉnh Quảng Trị năm 2018-2019

Đã gửi bởi xuanhoan23112002 on 06-06-2018 - 07:55 trong Tài liệu - Đề thi

Câu 6:

Từ giả thiết kết hợp với công thức khai triển bậc 4: $(a+b)^4=a^4+4a^3b+6a^2b^2+4ab^3+b^4$ ta có:

$Q=(x+3-x)^4-4x(3-x)(x^2+(3-x)^2)=81-2(9-x^2-(3-x)^2)(x^2+(3-x)^2)=81+2(x^2+(3-x)^2)^2-18(x^2+(3-x)^2)=2(x^2+(3-x)^2-5)^2+2(x^2+(3-x)^2)+31\geq 10+31=41$

Đẳng thức xảy ra  $$\Leftrightarrow x^2+(3-x)^2=5\Leftrightarrow 2x^2-6x+4=0\Leftrightarrow x=1, x=2$$

Vậy $Min Q=41\Leftrightarrow x=1, x=2$




#683639 Đề thi toán vòng 2 thpt chuyên Lê Hồng Phong Nam Định 2017-2018

Đã gửi bởi xuanhoan23112002 on 08-06-2017 - 09:54 trong Tài liệu - Đề thi

minh nghi bai to hop cac ban can chung minh co 1 tam giac co 3 dinh la 3 trong cac diem da cho va canh cua tam giac nho hon 1 thi dung cong thuc tinh dien tich bang sin60 se ra ngay




#683641 Đề thi toán vòng 2 thpt chuyên Lê Hồng Phong Nam Định 2017-2018

Đã gửi bởi xuanhoan23112002 on 08-06-2017 - 09:57 trong Tài liệu - Đề thi

Mình thấy cách giải của bạn ddang00 không hợp lí lắm. Như cách giải thích của anh IHateMath thì có vẻ cách của bạn chưa đúng hơn nữa nếu làm như vậy con số $\frac{3\sqrt{3}}{4}cm^{2}$ không có ý nghĩa cho lắm.

 

Cách giải của mình như sau:

 

attachicon.gifScreenHunter_35 May. 30 14.25.jpg

Ta chia tứ giác $ABCD$ thành $16$ tứ giác nội tiếp trong đường tròn bán kính $1$ như hình trên bằng cách lấy các trung điểm của cạnh tứ giác $ABCD$ và làm thế 1 lần nữa với $4$ tứ giác vừa được chia ra.

Theo nguyên lí $Dirichlet$ thì tồn tại $3$ điểm trong $33$ đã cho cùng thuộc $1$ tứ giác trong $16$ tứ giác vừa được chia ra

$3$ điểm này thuộc hình tròn bán kính bằng $1$. Ta sẽ chứng minh $3$ điểm này là $3$ điểm cần tìm.

attachicon.gifScreenHunter_36 May. 30 14.38.jpg

Gọi $3$ điểm này là $E,F,G$

Xảy ra $3$ trường hợp:

TH1 3 điểm này không nằm trên đường tròn. Vẽ $EF$ cắt $(I)$ tại $M$. Đường thẳng $EG$ cắt $(I)$ tại $N,K$.

Dễ thấy $S_{EFG}< S_{MNK}$. Mà ta lại có diện tích của 1 tam giác bất kì nội tiếp đường tròn không quá diện tích tam giác đều nội tiếp đường tròn đó. Dễ tính được diện tích tam giác đều nội tiếp đường tròn bán kính $1$ là $\frac{3\sqrt{3}}{4}cm^{2}$

Suy ra $S_{EFG}< S_{MNK}\leq \frac{3\sqrt{3}}{4}cm^{2} $

TH2 Tồn tại ít nhất $1$ điểm trong $3$ điểm nằm trên đường tròn.

Vẽ như TH1 và giải như TH1

TH3 3 điểm này nằm trên đường tròn. Giải như TH1 thì $S_{EFG}\leq  \frac{3\sqrt{3}}{4}cm^{2}$

 

Như vậy ta có điều phải chứng minh

sao ban biet 16 tu giac deu noi tiep




#683636 Đề thi toán vòng 2 thpt chuyên Lê Hồng Phong Nam Định 2017-2018

Đã gửi bởi xuanhoan23112002 on 08-06-2017 - 09:52 trong Tài liệu - Đề thi

moi nguoi hom nay co ket qua LHP day hoi hop qua hi vong do co vu cho minh nhe




#685108 Đề thi toán chuyên - chuyên KHTN ĐHQG HÀ Nội vòng 2 2017

Đã gửi bởi xuanhoan23112002 on 20-06-2017 - 09:12 trong Tài liệu - Đề thi

Câu II 2.

Từ giả thiết ta có được $\frac{1}{(a+1)(b+1)}+\frac{1}{(c+1)(b+1)}+\frac{1}{(a+1)(c+1)}=1$

 Đặt $a+1=\frac{\sqrt{3}}{x}, b+1=\frac{\sqrt{3}}{y},c+1=\frac{\sqrt{3}}{z}$

Giả thiết trở thành $xy+yz+zx=3$ và

$P= \sqrt{3} ( \frac{1}{\frac{3}{x}+x} +\frac{1}{\frac{3}{y}+y} +\frac{1}{\frac{3}{z}+z})$

   $= \sqrt{3} (\frac{x}{x^{2}+3}+\frac{y}{y^{2}+3}+\frac{z}{z^{2}+3})$

Sử dụng giả thiết ta có

  $P=\sqrt{3}( \frac{x}{(x+y)(x+z)}+ \frac{y}{(x+y)(y+z)}+ \frac{z}{(z+y)(x+z)})$

    $=\sqrt{3}( \frac{2(xy+yz+zx)}{(x+y)(y+z)(z+x)})$

Mặt khác $(x+y)(y+z)(z+x) \geq \frac{8}{9}(x+y+z)(xy+yz+zx) \geq \frac{8}{3}(xy+yz+zx)$

Suy ra $P \leq \sqrt{3}\frac{3}{4}= \frac{3\sqrt{3}}{4}$

Đẳng thức xảy ra khi $a=b=c = \sqrt{3}-1$                                                      

mình có cách khác nhanh hơn đặt a+1=x;b+1=y;c+1=z thì bài toán sẽ tự nhiên hơn và ứng dụng được cả giả thiết




#709817 Đề thi lớp 10 môn Toán vào Trường THPT Chuyên Lam Sơn

Đã gửi bởi xuanhoan23112002 on 03-06-2018 - 10:57 trong Tài liệu - Đề thi

Câu 5: Bất đẳng thức đã cho tương đương với:

$a^2b+b^2c+c^2a+ab^2+bc^2+ca^2> a^3+b^3+c^3+2abc$

$\Leftrightarrow a(b-c)^2+b^2(c+a-b)+c^2(a+b-c)> 0$ (luôn đúng đó a, b, c là độ dài 3 cạnh của 1 tam giác)




#712416 Đề thi IMO 2018

Đã gửi bởi xuanhoan23112002 on 12-07-2018 - 21:52 trong Thi HSG Quốc gia và Quốc tế

Kết quả chính thức IMO 2018



#707774 ĐỀ THI OLYMPIC CHUYÊN KHTN 2018

Đã gửi bởi xuanhoan23112002 on 06-05-2018 - 16:28 trong Thi HSG cấp Tỉnh, Thành phố. Olympic 30-4. Đề thi và kiểm tra đội tuyển các cấp.

2 ngày thì mọi người làm đc mấy bài




#705302 ĐỀ THI OLYMPIC 30/4 NĂM 2018 THPT LHP TP.HCM - KHỐI 10

Đã gửi bởi xuanhoan23112002 on 08-04-2018 - 23:13 trong Thi HSG cấp Tỉnh, Thành phố. Olympic 30-4. Đề thi và kiểm tra đội tuyển các cấp.

Bài 4:

Theo đề bài ta có: $p^2 -p+1=x^3$(x là số tự nhiên,x>1)

Hay $p(p-1)=(x-1)(x^2+x+1)$

Do $p$ là số nguyên tố nên $x-1$ hoặc $x^2+x+1$ chia hết cho$ p$

Nếu $x-1$ chia hết cho $p$ thì $x-1\ge p,x^2+x+1<p$ (vô lí với x là số tự nhiên >1) 

Do đó $x^2+x+1$ chia hết cho $p$ nên $x^2+x+1=pk$ (k là số tự nhiên)

Ta xét$ k=1,2$ không thỏa mãn

Xét $k\ge 3$

Thay vào phương trình ta được:$ p-1=(x-1)k$ hay $p=(x-1)k+1$

Từ đó ta có: $x^2+x+1=(xk-k+1)k$

Hay $x^2+x(1-k^2)+k^2-k+1=0$

Đây là phương trình bậc 2 ẩn x, để phương trình có nghiệm tự nhiên thì $\delta =k^4-6k^2+4k-3$ phải là số chính phương

Ta có: $(k^2-3)^2\le k^4-6k^2+4k-3<(k^2-2)^2$

Từ đó ta tìm được k=3 ta tìm được x=7, p=19 là số nguyên tố

Vậy p=19 là số nguyên tố thỏa mãn đề bài




#705345 Cần lắm một lời giải thích !

Đã gửi bởi xuanhoan23112002 on 09-04-2018 - 20:48 trong Hình học phẳng

Ý mk hỏi [mod $\pi$] nghĩa là gì

góc định hướng giữa 2 đường thẳng hơn nhau 1 bội của π

 




#709300 Bất đẳng thức trong đề thi vào lớp 10 Nam Định năm 2018

Đã gửi bởi xuanhoan23112002 on 26-05-2018 - 17:55 trong Bất đẳng thức và cực trị

Đây là đề của ban xã hội bạn ạ.

Còn đây là lời giải của mình cho bài toán này các bạn có thể tham khảo:

Coi phương trình trên là phương trình bậc 2 ẩn a theo công thức nghiệm ta được

$a=\frac{-bc+\sqrt{(4-b^2)(4-c^2)}}{2}$

Áp dụng bất đẳng thức Cauchy cho căn thức trong biểu thức trên, ta có:

$a\leq \frac{-bc+\frac{4-b^2+4-c^2}{2}}{2}= \frac{8-(b+c)^2}{4}$

Từ đó ta có: $2a+b+c=\frac{8-(b+c)^2+2(b+c)}{2}=\frac{9-(b+c-1)^2}{2}\leq \frac{9}{2}$

P/s: Mình nghĩ đây là cách ngắn nhất và có thể thay số 2 trong đề bài bởi các số khác vẫn có thể giải tương tự.




#709289 Bất đẳng thức trong đề thi vào lớp 10 Nam Định năm 2018

Đã gửi bởi xuanhoan23112002 on 26-05-2018 - 15:38 trong Bất đẳng thức và cực trị

Cho a, b, c là các số thực dương thỏa mãn: $a^2+b^2+c^2+abc=4$. Chứng minh rằng:

$2a+b+c\leq \frac{9}{2}$




#709502 Bất đẳng thức chọn lọc ôn chuyên

Đã gửi bởi xuanhoan23112002 on 29-05-2018 - 15:47 trong Bất đẳng thức và cực trị

Bài 3: Theo giả thiết ta có $0\leq a, b, c\leq 4$ nên

$$(4-a)(4-b)(4-c) \geq 0$$

$\Leftrightarrow 64+4(ab+bc+ca) \geq abc+16(a+b+c)$

$\Leftrightarrow ab+bc+ca\geq 8+\frac{abc}{4}\geq 8$ 

Do đó ta có: $P=(a+b+c)^2-(ab+bc+ca)\leq 36-8=28$

Đẳng thức xảy ra $\Leftrightarrow (a, b, c)=(0, 2, 4)$ và các hoán vị của nó

Vậy $MaxP=28$ $\Leftrightarrow (a, b, c)=(0, 2, 4)$ và các hoán vị của nó




#707928 Bất đẳng thức

Đã gửi bởi xuanhoan23112002 on 08-05-2018 - 21:35 trong Bất đẳng thức và cực trị

BĐT$\Leftrightarrow \frac{a^2}{ab+ac}+\frac{b^2}{bc+bd}+\frac{c^2}{cd+ca}+\frac{d^2}{da+db}\geq 2$

Ta có VT$\geq \frac{(a+b+c+d)^2}{(a+d)(b+c)+(c+d)(a+b)}$( theo BĐT Cauchy-Schwarz)

Mà cũng theo BĐT AM-GM ta cũng có $(a+d)(b+c)+(a+b)(c+d)\leq \frac{(a+b+c+d)^2}{2}$

Do đó VT$\geq 2$

Vậy bất đẳng thức được chứng minh. Đẳng thức xảy ra$\Leftrightarrow$ $a= b= c= d> 0$




#709807 Bất đẳng thức

Đã gửi bởi xuanhoan23112002 on 03-06-2018 - 07:54 trong Bất đẳng thức - Cực trị

Cho a, b, c là các số thực không âm. Chứng minh rằng:

$3(a+b+c)\geq 2(\sqrt{a^2+bc}+\sqrt{b^2+ca}+\sqrt{c^2+ab})$




#709479 Bài tập về đa thức

Đã gửi bởi xuanhoan23112002 on 29-05-2018 - 08:53 trong Đa thức

Bài 1: Cho đa thức $f(x)=x^{2018}+\sum a_ix^{i}($a_i\in {-1,1}, $\forall i\in \left \{ 0,1,...,2017 \right \}$$)$ không có nghiệm thực. Tìm số lớn nhất các hệ số = -1 trong f(x)

Bài 2: Tìm đa thức P(x) hệ số thực thỏa mãn:

$(P(x))^{3}-3(P(x))^{2}=P(x^{3})-3P(-x)$, với mọi x là số thực




#710198 AP vuông góc với IJ

Đã gửi bởi xuanhoan23112002 on 07-06-2018 - 14:17 trong Hình học

Cho tam giác ABC với AC > AB. Các đường cao BB, CCcủa tam giác cắt nhau tại H. Gọi M, N lần lượt là trung điểm của BC', CB'. MH cắt đường tròn ngoại tiếp tam giác CHB' tại I; N

H cắt đường tròn ngoại tiếp tam giác BHC' tại J. Giả sử P là trung điểm cạnh BC. Chứng minh: AP vuông góc với IJ




#688311 58th IMO 2017

Đã gửi bởi xuanhoan23112002 on 22-07-2017 - 08:38 trong Thi HSG Quốc gia và Quốc tế

moi nguoi nghi ra huong giai quyet bai 3 chua




#703960 2(ab+bc+ca)+$\frac{1}{ab}+\frac{1...

Đã gửi bởi xuanhoan23112002 on 19-03-2018 - 23:21 trong Bất đẳng thức và cực trị

Su dung pp pqr
Dat p=a+b+c=3

q=ab+bc+ca

r=abc,r<=1

BDT tuong duong 2q+3/r>=9

Hay 2qr+3>=9r

Ma q>=3*can(r)( do q^2>=3pr)

Dua ve bpt an r giai voi chu y r<=1